Question 3 The position of an object as a function of time is given by z(t) = (4.1m/s³)t³ - (4.0m/s²)t² + (55m/s)t - 7.0m Part A Find the instantaneous acceleration at t = 3.4s Express your answer

Answers

Answer 1

The instantaneous

acceleration

at t = 3.4s is 75.64 m/s².

To find the instantaneous acceleration at a specific

time

, we need to differentiate the position function with respect to time twice.

Given:

z(t) = (4.1m/s³)t³ - (4.0m/s²)t² + (55m/s)t - 7.0m

First, let's differentiate z(t) with respect to time to find the

velocity

function:

v(t) = d/d (z(t))

v(t) = d/dt((4.1m/s³)t³ - (4.0m/s²)t² + (55m/s)t - 7.0m)

To differentiate each term, we apply the

power

rule:

d/dt(tⁿ) = n(t^(n-1))

v(t) = (4.1m/s³)(3t²) - (4.0m/s²)(2t) + 55m/s

Simplifying further:

v(t) = 12.3t² - 8.0t + 55m/s

Now, let's differentiate v(t) with respect to time to find the acceleration function:

a(t) = d/dt(v(t))

a(t) = d/dt(12.3t² - 8.0t + 55m/s)

Again, applying the power rule:

a(t) = 2(12.3t) - 8.0

Simplifying further:

a(t) = 24.6t - 8.0

Now, we can substitute t = 3.4s to find the instantaneous acceleration at t = 3.4s:

a(3.4) = 24.6(3.4) - 8.0

a(3.4) = 83.64 - 8.0

a(3.4) = 75.64 m/s²

Therefore, the

instantaneous

acceleration at t = 3.4s is 75.64 m/s².

To know  more about

acceleration

visit:

https://brainly.com/question/460763

#SPJ11


Related Questions

An electron is circularly orbiting a proton. The magnitude of
acceleration of the electron is 9.17E8 m/s2. What is the
electron’s orbital radius (in meters)?

Answers

The electron’s orbital radius  5.2486 x 10^-4 meters.

The acceleration of an electron in a circular orbit can be related to its orbital radius (r) and the magnitude of the electrostatic force between the electron and the proton.

The electrostatic force between the electron and the proton can be given by Coulomb's law:

F = k * (|q1| * |q2|) / r^2

Where:

F is the electrostatic force,

k is Coulomb's constant (approximately 8.99 x 10^9 N·m^2/C^2),

|q1| and |q2| are the magnitudes of the charges (charge of an electron and charge of a proton are equal but opposite, |q1| = |q2| = 1.6 x 10^-19 C),

r is the orbital radius.

The magnitude of the acceleration of the electron (a) is related to the electrostatic force (F) by Newton's second law:

F = m * a

Where:

m is the mass of the electron (approximately 9.1 x 10^-31 kg).

By equating these two equations, we can find the orbital radius (r).

k * (|q1| * |q2|) / r^2 = m * a

r = √((k * (|q1| * |q2|)) / (m * a))

Substituting the given values:

r = √((8.99 x 10^9 N·m^2/C^2 * (1.6 x 10^-19 C)^2) / (9.1 x 10^-31 kg * 9.17 x 10^8 m/s^2))

r=5.2486 x 10^-4 meters.

Learn more acceleration:https://brainly.com/question/460763

#SPJ11

The magnetic field lines produced by a straight wire with a current entering the paper are:

a. in the parallel direction of the current
b. parallel but in the opposite direction to the current
c. exit the wire in a radially outward direction
d. in concentric circles clockwise around the wire
F. in concentric circles counterclockwise around the wire

Answers

The magnetic field lines produced by a straight wire with a current entering the paper are exit the wire in a radially outward direction. Option (C) is correct.

The direction of magnetic field lines produced by a straight wire with a current entering the paper is such that they exit the wire in a radially outward direction. This indicates that the magnetic field is directed in a clockwise direction when viewed from the opposite end.

The concentric circles formed around the wire are counterclockwise, which indicates that the magnetic field is also directed in a counterclockwise direction when viewed from above the wire.

The magnetic field lines produced by a straight wire with a current entering the paper can be determined using the right-hand rule. Curl your right hand such that your fingers point in the direction of the current, and your thumb will point in the direction of the magnetic field.

Know more about magnetic field, here:

https://brainly.com/question/14848188

#SPJ11

Consider a 5 solar mass red giant star with high metallicity. Which of the following is true? (choose all that apply)
O It is probably a member of Population I.
O It is likely to be found in a globular cluster.
O It could not be located on a spiral arm.
O It is composed of primordial material.

Answers

A red giant star with high metallicity and a mass of 5 solar masses is likely a member of Population I and could be located on a spiral arm. Option C is correct answer.

A red giant star with high metallicity, such as the one described, is more likely to be a member of Population I. Population I stars are typically younger stars found in the disk of a galaxy, and they have higher metallicity due to their formation from enriched interstellar material.

Globular clusters, on the other hand, consist mainly of Population II stars, which are older and have lower metallicity. Therefore, it is unlikely to find the 5 solar mass red giant star in a globular cluster.

As for its location on a spiral arm, it is possible for the star to be located there. Spiral arms of galaxies are regions with a higher density of stars and gas, making them favorable environments for star formation.

The composition of the star cannot be determined solely based on its mass and metallicity. Whether it is composed of primordial material or enriched material from previous stellar generations would require further information about its formation and evolutionary history.

Learn more about globular cluster here

https://brainly.com/question/31544394

#SPJ11

The complete question is

Consider a 5 solar mass red giant star with high metallicity. Which of the following is true? (choose all that apply)

A It is probably a member of Population I.

B It is likely to be found in a globular cluster.

C It could not be located on a spiral arm.

D It is composed of primordial material.

Which best describes how wave intensity and loudness are typically related?

Answers

While wave intensity provides a measurable physical quantity, loudness reflects the subjective perception of sound intensity and is influenced by various factors beyond wave intensity alone.

In the context of sound, wave intensity and loudness are closely related but not directly interchangeable terms. Wave intensity refers to the physical measure of the energy carried by a sound wave per unit area. It is quantified in terms of power per unit area and is typically measured in watts per square meter (W/m²). On the other hand, loudness is a subjective perception of the intensity of a sound experienced by a human ear.

While wave intensity and loudness are related, the perception of loudness is influenced by various factors, including the sensitivity of the human ear to different frequencies and the individual's threshold of hearing. The relationship between wave intensity and loudness follows a logarithmic scale known as the decibel (dB) scale.

The decibel scale allows us to quantify and compare different sound intensities based on their relative loudness to the human ear. It is important to note that a doubling of wave intensity does not necessarily result in a doubling of perceived loudness. Instead, a 10-fold increase in wave intensity corresponds to approximately a perceived doubling of loudness.

For such more questions on intensity

https://brainly.com/question/4431819

#SPJ8

Note- The complete question is "Which best describes how wave intensity and loudness are typically related in the context of sound?"

Drag force on a body increases with the increase in speed of the body . Justify with one example from everyday experience

Answers

Explanation:

One everyday example that justifies the increase in drag force with the increase in speed is riding a bicycle. When you ride a bicycle, you can feel the resistance against your body as you increase your speed.

At low speeds, the drag force is relatively low. However, as you start pedaling faster and gain speed, you will notice an increasing resistance against your body. This resistance is caused by the drag force acting on you as you move through the air.

As you accelerate on a bicycle, the air molecules in front of you get compressed, creating an area of high pressure. Simultaneously, the air molecules behind you expand, creating an area of low pressure. This pressure difference creates a force that opposes your forward motion, known as drag force or air resistance.

At higher speeds, the drag force becomes more significant and requires more effort to overcome. This is why you need to pedal harder or lean forward to reduce your frontal area and decrease the resistance as you ride faster.

The experience of feeling increased resistance while cycling at higher speeds demonstrates the effect of drag force increasing with speed. This principle applies not only to bicycles but also to various other objects moving through a fluid medium, such as cars, airplanes, or even a person running against the wind.

I hope I helped



A ball is shot from the ground straight up into the air with an initial velocity of 46 ft/sec. Assuming that the air resistance can be ignored, how high does it go?
Equations of Motion:

Recall that when we have linear motion that is undergoing constant acceleration
, then we can write a position function as

Answers

The ball reaches a maximum height of 46.94 feet when shot straight up into the air from the ground with an initial velocity of 46 ft/sec. This can be calculated using the equations of motion for linear motion with constant acceleration.

To determine the maximum height, we need to find the time it takes for the ball to reach its highest point. Since the ball is shot straight up, its velocity decreases at a constant rate due to the acceleration of gravity (-32 ft/sec²). The time it takes for the ball to reach its highest point can be found using the equation:

[tex]\[v_f = v_i + at\][/tex]

where [tex]\(v_f\)[/tex] is the final velocity (0 ft/sec at the highest point), [tex]\(v_i\)[/tex] is the initial velocity (46 ft/sec), a is the acceleration (-32 ft/sec²), and t is the time.

Substituting the known values into the equation, we can solve for t:

[tex]\[0 = 46 - 32t\][/tex]

Simplifying the equation, we get:

[tex]32t = 46 \\t = \frac{46}{32} = 1.4375 \text{ seconds}[/tex]

Now that we know the time it takes to reach the highest point, we can calculate the maximum height by using the equation:

[tex]\[h = v_i t + \frac{1}{2} a t^2\][/tex]

Substituting the known values into the equation, we get:

[tex]\[h = 46 \times 1.4375 + \frac{1}{2} (-32) \times (1.4375)^2\][/tex]

Simplifying the equation, we find:

[tex]\[h = 46.94 \text{ feet}\][/tex]

Therefore, the ball reaches a maximum height of approximately 46.94 feet.

To learn more about acceleration refer:

https://brainly.com/question/30595126

#SPJ11

Create a 50-word summary, in your own words, that describes how accurately the circumference of Earth can be measured using the Sun. You should cite specific evidence you have collected in your description. Feel free to reference things learned in specific parts (I and II), or to create label sketches to illustrate your response.

Answers

The circumference of the Earth can be accurately measured by observing the Sun. Eratosthenes was an ancient Greek astronomer and mathematician who used this method to estimate the Earth's circumference in the third century BC.

He noticed that on the summer solstice, the Sun shone directly into a deep well in Syene, Egypt while casting a shadow on a stick in Alexandria, about 800 km to the north. By measuring the angle of the shadow in Alexandria, he was able to determine the angle between the Sun and the zenith, which he found to be 1/50th of a circle. He then multiplied this angle by the distance between the two cities to estimate the circumference of the Earth, which he found to be around 39,375 km. This method is still used today, with modern instruments allowing for even greater accuracy.

To know more about circumference visit:

https://brainly.com/question/28757341

#SPJ11

Sam heaves a 16 lblb shot straight upward, giving it a constant upward acceleration from rest of 36.0 m/s2m/s2 for a height 68.0 cmcm. He releases it at height 2.30 mm above the ground. Ignore air resistance.

A) What is the speed of the shot when he releases it?Express your answer with the appropriate units.

B) How high above the ground does it go?Express your answer with the appropriate units.

C) How much time does he have to get out of its way before it returns to the height of the top of his head, a distance 1.73 mm above the ground?

Express your answer with the appropriate units.

Answers

The time taken by the shot to reach the height of Sam's head is 0.196 s.

The initial velocity of the shot u = 0 m/s.

Acceleration, a = 36.0 m/s²

Displacement, s = 68 cm = 0.68 m

We can use the formula v² = u² + 2as, where v is the final velocity of the shot.

v² = u² + 2asv² = 0 + 2(36.0 m/s²)(0.68 m)

           = 49.536 m²/s²v = √(49.536 m²/s²) = 7.04 m/s

Therefore, the speed of the shot when Sam releases it is 7.04 m/s. (DETAIL ANS)

B) Let h be the maximum height the shot reaches above the ground.

Using the equation v² = u² + 2as, we can find the maximum height, h.

v² = u² + 2ash = (v² - u²) / (2a)h = (7.04 m/s)² / (2 × 36.0 m/s²) = 0.707 m

Therefore, the height that the shot reaches above the ground is 0.707 m. (DETAIL ANS)

C) Sam's height is 1.73 m above the ground.

We can find the time taken by the shot to reach the height of Sam's head using the equation v = u + at, where v is the final velocity, u is the initial velocity, a is the acceleration, and t is the time taken

.v = u + atv = 0 + (36.0 m/s²)t = 36.0tt = v / at = (7.04 m/s) / (36.0 m/s²) = 0.196 s

Therefore, the time taken by the shot to reach the height of Sam's head is 0.196 s. .

Learn more about acceleration

brainly.com/question/2303856

#SPJ11

to Assignment Bookdet C1 16. A balloon with a radius of 16 cm has an electric charge of 4.25 x 10°C. 2. Determine the electric field strength at a distance of 40.0 cm from the balloon's centre. b. A

Answers

A balloon with a radius of 16 cm has an electric charge of 4.25 x 10°C. The electric field strength at a distance of 40.0 cm from the balloon's center is approximately 1.524 x 10^4 N/C.

To determine the electric field strength at a distance of 40.0 cm from the balloon's center, we can use Coulomb's law. Coulomb's law states that the electric field strength (E) at a point is directly proportional to the magnitude of the electric charge (Q) and inversely proportional to the square of the distance (r) from the charge.

The formula for the electric field strength is:

E = k * (Q / r^2)

where:

E is the electric field strength,

k is the electrostatic constant (k ≈ 8.99 x 10^9 Nm^2/C^2),

Q is the electric charge, and

r is the distance from the charge.

Given:

Radius of the balloon (r) = 16 cm = 0.16 m

Electric charge (Q) = 4.25 x 10^(-6) C

Distance from the balloon's center (r) = 40.0 cm = 0.40 m

Let's calculate the electric field strength (E):

E = k * (Q / r^2)

E = (8.99 x 10^9 Nm^2/C^2) * (4.25 x 10^(-6) C / (0.40 m)^2)

E = (8.99 x 10^9 Nm^2/C^2) * (4.25 x 10^(-6) C / 0.16 m^2)

E = (8.99 x 10^9 Nm^2/C^2) * (4.25 x 10^(-6) C / 0.0256 m^2)

E = (8.99 x 10^9 N * 4.25 x 10^(-6)) / 0.0256 m^2

E = 1.524 x 10^4 N/C

Therefore, the electric field strength at a distance of 40.0 cm from the balloon's center is approximately 1.524 x 10^4 N/C.

To learn more about electric field visit: https://brainly.com/question/19878202

#SPJ11

At distance r from a point charge q, the electric potential is 883 V and the magnitude of the electric field is 300 N/C. Determine the value of q. The Coulomb constant is 8.98755 × 109 N · m2 /C2 . Answer in units of C.

Answers

At a distance of r from a point charge q, the electric potential is 883 V and the magnitude of the electric field is 300 N/C.

We are supposed to determine the value of q, and the Coulomb constant is 8.98755 × 109 N·m²/C².

The electric potential V at a distance r from the point charge is given by the formula V = kq/r,

where k = 8.98755 × 10^9 N·m²/C².

Putting the values, V = kq/r

r = kq/V.

According to the problem, V = 883 V and r is not given.

So, we will write V in terms of E.

Electric potential V is related to electric field E by the formula

V = Er.

Therefore, 883 = E r....(1)

According to the problem, E = 300 N/C.

Substituting E in equation (1), we get 883 = 300 × r

r = 2.943 m

Now we can use the formula for electric field

E = kq/r²

Substituting the values of k, q, and r, we have

300 = [8.98755 × 10^9 × q]/[2.943]²q

= [300 × 2.943²] / 8.98755 × 10^9

= 2.775 × 10^-7 C.

So the value of q is 2.775 × 10^-7 C.

To learn more about distance visit;

https://brainly.com/question/13034462

#SPJ11

A wind turbine with a blade diameter of 25 m is to be installed in a location where average wind velocity is 6 m/s. If the overall efficiency of the turbine is 34 percent, determine (a) the average electric power output, (b) the amount of electricity produced from this turbine for an annual operating hours of 8000 h, and (c) the revenue generated if the electricity is sold at a price of $0.09/kWh.Take the density of air to be 1.3 kg/m'.

Answers

The wind turbine with a blade diameter of 25 m and an average wind velocity of 6 m/s has an average electric power output of 172.34 kW. For an annual operating time of 8000 hours, the turbine will produce approximately 1,378,720 kWh of electricity.

If sold at a price of $0.09/kWh, the revenue generated from the electricity produced by the turbine would be approximately $124,086.72. To calculate the average electric power output, we can use the formula:

[tex]\[P = \frac{1}{2} \times \text{{density of air}} \times A \times v^3 \times \text{{efficiency}}\][/tex]

Substituting the given values into the formula, we can calculate the average electric power output:

[tex]\[P = \frac{1}{2} \times 1.3 \, \text{{kg/m}}^3 \times \left(\frac{\pi}{4} \times (25 \, \text{{m}})^2\right) \times (6 \, \text{{m/s}})^3 \times 0.34 \approx 172340 \, \text{{watts}} \approx 172.34 \, \text{{kW}}\][/tex]

To determine the amount of electricity produced for an annual operating time of 8000 hours, we multiply the average electric power output by the operating time:

[tex]\[\text{{Electricity produced}} = P \times \text{{operating time}} = 172.34 \, \text{{kW}} \times 8000 \, \text{{h}} = 1,378,720 \, \text{{kWh}}\][/tex]

Finally, to calculate the revenue generated, we multiply the electricity produced by the selling price per kilowatt-hour:

[tex]\[\text{{Revenue}} = \text{{Electricity produced}} \times \text{{price per kWh}} = 1,378,720 \, \text{{kWh}} \times \$0.09/\text{{kWh}} \approx \$124,086.72\][/tex]

To learn more about power refer:

https://brainly.com/question/1634438

#SPJ11

Air enclosed in a cylinder has density = 1. 4 kg/m3. A. What will be the density of the air if the length of the cylinder is doubled while the radius is unchanged?=______________kg/m3b. What will be the density of the air if the radius of the cylinder is halved while the length is unchanged?

Answers

a) Density of air is ρ = m/V (where m is the mass of air enclosed). As the mass of air enclosed remains constant, so ρ'=ρ/2 ; b) New density of air when the length of cylinder is doubled while the radius is unchanged is 0.7 kg/m³.

a. Let's assume the volume of air is V initially and the length of the cylinder is L and radius is R. Now, the new length will be 2L

keeping the radius R. As the volume of the cylinder is given as V = πR²L, the new volume can be written as

V' = πR²(2L).

Hence, the density of air can be written as ρ = m/V (where m is the mass of air enclosed). As the mass of air enclosed remains constant, ρ'=ρ/2

b. Let's assume the initial radius of the cylinder as R. Now, the radius is halved, which means the new radius will be R/2, while the length of the cylinder is unchanged, i.e., L. The new volume can be written as

V' = π(R/2)²L

= πR²L/4.

Now, the density of air can be written as ρ = m/V (where m is the mass of air enclosed). As the mass of air enclosed remains constant, ρ' = ρ*4

We know that Density of air enclosed in the cylinder is given as ρ= 1.4 kg/m³

Now, the volume of cylinder is given as V = πr²h

Given volume is V, we need to find the density when the length of the cylinder is doubled while the radius is unchanged.

New length of the cylinder = 2h (doubled), New volume of the cylinder = πr² (2h)

= 2πr²h

We know that Density is given by ρ = m/V

Given, ρ = 1.4 kg/m³, Initial volume = V, Initial mass of the air enclosed

= m

Let the new density be ρ'.

Now, ρ' = m/2πr²h

We can write the above equation as: ρ' = ρ (V/2V)

= 0.5 ρ

The new density of air when the length of the cylinder is doubled while the radius is unchanged is 0.7 kg/m³.

To know more about density, refer

https://brainly.com/question/26364788

#SPJ11

An elevator weighing 2500 N ascends at a constant speed of 8.0 m/s. How much power must the motor supply to do this?

Answers

The power required to lift the elevator is equal to the weight of the elevator multiplied by the speed at which it is ascending. The power required by the elevator motor to ascend at a constant speed is 20,000 W.

To calculate the power required by the elevator motor to ascend at a constant speed, we need to use the formula

P = Fv,

where,

P is power,

F is force,

v is velocity.

Given that the elevator weighs 2500 N and is ascending at a constant speed of 8.0 m/s, we can calculate the power required by the elevator motor as follows:

Force = 2500 N

Velocity = 8.0 m/s

Power = Force x Velocity

= 2500 N x 8.0 m/s

= 20,000 W

20KW

Therefore, the power required by the elevator motor to ascend at a constant speed is 20,000 W.

To know more about power:

https://brainly.com/question/31481646

#SPJ12

Use Newton's Method to find the critical value of f(x) = xsin (x) on the interval (0,7). Repeat the method until your estimates change by less than one one-thousandth of a unit. Your solution must include the formula used for this function. This is the only problem for which you may use a calculator.

Answers

The critical value of f(x) = xsin(x) on the interval (0, 7) is 4.903 (rounded to 3 decimal places).

Newton's Method to find the critical value of f(x) = xsin(x) on the interval (0,7) is given by;

Let the initial estimate for the critical point in the interval (0, 7) be x1.

Newton's method formula is given by: x2 = x1 - f(x1)/f'(x1)The formula for this function is; `f(x) = xsin(x)`where f'(x) = sin(x) + x cos(x).

Thus, the Newton's method formula for this function is;x2 = x1 - (x1 sin(x1)) / (sin(x1) + x1 cos(x1))Given interval (0, 7).Taking x1 = 5,x2 = 5 - (5sin(5)) / (sin(5) + 5cos(5))= 4.88626 (rounded to 5 decimal places)Taking x1 = 4.88626,x2 = 4.88626 - (4.88626sin(4.88626)) / (sin(4.88626) + 4.88626cos(4.88626))= 4.90322 (rounded to 5 decimal places)

Taking x1 = 4.90322,x2 = 4.90322 - (4.90322sin(4.90322)) / (sin(4.90322) + 4.90322cos(4.90322))= 4.90318 (rounded to 5 decimal places)

Therefore, the critical value of f(x) = xsin(x) on the interval (0, 7) is 4.903 (rounded to 3 decimal places).

To learn more about interval visit;

https://brainly.com/question/11051767

#SPJ11

A mug of tea has been left to cool from 90 C to 18 C. If there
is 0.2 kg of tea in the mug, and the tea has specific heat capacity
4200 J K−1 kg−1, calculate the change in
entropy of the tea.

Answers

A mug of tea has been left to cool from 90 C to 18 C. If there is 0.2 kg of tea in the mug, and the tea has specific heat capacity 4200 J K−1 kg−1, the change in entropy of the tea is 5040 J/K.

The formula for entropy can be given as:

S = Q / T

Where,

S is the entropy,

Q is the heat transferred,

T is the temperature.

A mug of tea has been left to cool from 90 C to 18 C. If there is 0.2 kg of tea in the mug, and the tea has specific heat capacity 4200 J K−1 kg−1, the change in entropy of the tea is:

ΔS = (Q / T) = (mcΔT / T)

Where,

m is the mass,

c is the specific heat capacity,

ΔT is the change in temperature of the tea

ΔS = (mcΔT / T) = (0.2 x 4200 x (90 - 18) / 18) = 5040 J/K

Learn more about specific heat capacity:

https://brainly.com/question/27991746

#SPJ11

A magnetic field cannot:
a. exert a force on a load
b. speed up a load
C. change the momentum of a charge
d. change the kinetic energy of a charge

Answers

A magnetic field cannot change the kinetic energy of a charge. Option (D) is correct.

A magnetic field is a region in which a magnetic force can be observed. It is a consequence of the motion of electric charges. Magnetic fields can exert a force on moving charged particles, deflecting them from their initial trajectory and changing their direction.

It can also change the direction of the magnetic moment of an atomic-scale magnet or cause a torque to act on it. A magnetic field cannot, however, change the kinetic energy of a charged particle. This is because the magnetic force on a moving charge is perpendicular to its velocity and does no work on the particle, hence it does not change its kinetic energy.

The magnetic force alters the direction of motion of a charge and causes it to move in a circular path. The magnetic force only changes the direction of the velocity of the charged particle, not the magnitude of its velocity.

Know more about kinetic energy, here:

https://brainly.com/question/999862

#SPJ11

Performance Task 2 (Final term) Volume of Solids 1. A wooden rectangular hollow box has outer edges of 6 in and 8 in. and height of 10 in. The uniform thickness of the box is 1 in. a. Find the amount

Answers

Final-term performance task no. 2 Volume of Solids 1. A hardwood rectangular hollow box with 10-inch height and 6-inch and 8-inch outer borders. The uniform thickness of the box is 1 in :

1. Wooden box: material used = 165 cubic inches, blocks that fit = 165

2. Water tank: volume of water = 120π cubic inches

3. Frustum: lateral area = 140 square meters, total surface area = 240 square meters, volume = 368π cubic inches

4. Hill: amount of earth removed = 220000 cubic meters

Here is the explanation :

1. a. The amount of material used is the difference between the outer volume and the inner volume. The outer volume is 6810 = 480 cubic inches. The inner volume is 579 = 315 cubic inches. The difference is 480-315 = 165 cubic inches.

b. The number of wooden blocks that will fit into the box is the volume of the box divided by the volume of a single block. The volume of the box is 165 cubic inches. The volume of a single block is 1 cubic inch. The number of blocks that will fit is [tex]\frac{165}{1}[/tex] = 165 blocks.

2. The volume of water in the tank is the volume of the cone times the fraction of the cone that is filled with water. The volume of the cone is [tex]\begin{equation}\frac{1}{3}\pi r^2 h = \frac{1}{3}\pi 6^2 12 = 288 \pi \text{ cubic inches}[/tex]. The fraction of the cone that is filled with water is [tex]\frac{5}{12}[/tex]. The volume of water in the tank is 288 pi * [tex]\frac{5}{12}[/tex] = 120 pi cubic inches.

3. a. The lateral area of the frustum is the area of the curved surface. The curved surface is made up of two triangles and a rectangle. The area of a triangle is ([tex]\frac{1}{2}[/tex])base height. The base of each triangle is 8 meters and the height is 10 meters. The area of each triangle is ([tex]\frac{1}{2}[/tex])810 = 40 square meters. The area of the rectangle is 610 = 60 square meters. The total area of the curved surface is 402 + 60 = 140 square meters.

b. The total surface area of the frustum is the area of the curved surface plus the area of the two bases. The area of the two bases is 8² = 64 square meters + 6² = 36 square meters = 100 square meters. The total surface area is 140 + 100 = 240 square meters.

c. The volume of the frustum is the volume of the whole pyramid minus the volume of the smaller pyramid. The volume of the whole pyramid is [tex]\begin{equation}\frac{1}{3}\pi r^2 h = \frac{1}{3}\pi 8^2 10 = 512 \pi \text{ cubic inches}[/tex]. The volume of the smaller pyramid is [tex]\begin{equation}\frac{1}{3}\pi(6^2)(10) = 144\pi \text{ cubic inches}[/tex]. The volume of the frustum is 512 pi - 144 pi = 368 pi cubic inches.

4. The amount of earth removed is the volume of the prismatoid. The volume of the prismatoid is the area of the lower base times the height plus the area of the right triangle times the height. The area of the lower base is 160100 = 16000 square meters. The area of the right triangle is ([tex]\frac{1}{2}[/tex])60200 = 6000 square meters. The height of the prismatoid is 10 meters. The volume of the prismatoid is 1600010 + 6000*10 = 220000 cubic meters.

To know more about the prismatoid refer here :

https://brainly.com/question/28192146#

#SPJ11

Complete question :

Performance Task 2 (Final term) Volume of Solids 1. A wooden rectangular hollow box has outer edges of 6 in and 8 in. and height of 10 in. The uniform thickness of the box is 1 in. a. Find the amount of the material used b. How many wooden blocks each cube with 1 in. edge will fit into the box? 2. A water tank is in the form of a right circular cone with a base radius 6 ft and a slant height of 12 ft. find the volume of the water when the tank is filled to a depth of 5 ft. 3. Consider a frustum of a regular pyramid. The upper base is a square of side equal to 6 meters while the lower base is also a square of side 8 meters and an altitude of 10 meters. Find the following: a. Lateral area b. Total surface area c. Volume 4. In a certain railroad landslide cleaning operation in Baguio there was a need to remove a portion of a hill which was in the shape of a prismatoid. The lower base is a rectangle and the upper base is a right triangle. GE | AB, EF AC and all face angles at E and A are right angles. The height of the solid is 10 m. find the amount of earth removed. 160 100 M G B 200 C 60

An unidentified flying object (UFO) is observed to travel a total distance of 19000 m, starting and ending at rest, over a duration of 4.41 s. Assuming the UFO accelerated at a constant rate to the midpoint of its journey and then decelerated at a constant rate the rest of the way, what was the magnitude of its acceleration? Express your answer in gs, where 1 g -9.81 m/s^2. 398 gs 199 gs 3.908 gs 1,954 g s

Answers

The magnitude of the UFO's acceleration is 398 gs, Answer: (a) .

Total distance traveled by UFO, s = 19,000 m

Time taken, t = 4.41 s

The UFO accelerated at a constant rate to the midpoint of its journey and then decelerated at a constant rate the rest of the way.

Using the formula:

s = ut + 1/2 at²

where,

s = distance traveled by UFO in m,

u = initial velocity in m/s,

a = acceleration in m/s²,

t = time taken in s

Let's assume the UFO took t/2 time to reach the midpoint of its journey.

Initial velocity of the UFO, u = 0m/s

Let's find the distance traveled by UFO during the first half of its journey:

d1 = ut + 1/2at²

for t/2 time,

d1 = 0 × t/2 + 1/2a(t/2)²

d1 = at²/8

d1 = a(4.41/2)²/8

d1 = a × 0.61096²/8

d1 = a × 0.093750536

d1 = 0.09375a(Equation 1)

Let's find the distance traveled by UFO during the second half of its journey:

d2 = ut + 1/2at²                 

for t/2 time,

d2 = v(t/2) - 1/2a(t/2)²

d2 = v × t/2 - 1/2a(t/2)²,

where,

v is the velocity of the UFO at the midpoint of its journey

d2 = (19000/2) - 1/2a(t/2)²

d2 = 9500 - 1/2a(4.41/2)²

d2 = 9500 - 1/2a × 0.61096²

d2 = 9500 - 0.09375a(Equation 2)

Let's add equations 1 and 2 to get the total distance traveled by the UFO:

s = d1 + d2 = 0.09375a + 9500 - 0.09375as = 9500m

As the UFO was at rest before and after traveling the distance, the final velocity of the UFO is also 0 m/s.

Using the formula:

s = (u + v)t/2

where,

s = 9500 m,

u = initial velocity in m/s,

v = final velocity in m/s,

t = time taken in s

t = 4.41 s

We know that the UFO accelerated for t/2 time and decelerated for t/2 time, so the final velocity of the UFO can be calculated using the formula:

v = u + at/2 for the second half of the journey.

Let's assume the magnitude of acceleration of the UFO is a g, where 1 g = 9.81 m/s²

Magnitude of acceleration of the UFO in m/s² = 9.81 × ag = 398g(since 1 g = 9.81 m/s²)

Hence, the magnitude of the UFO's acceleration is 398 gs.

Answer: (a) 398 gs

Learn more about acceleration:

https://brainly.com/question/25876659

#SPJ11

1.00 kg of ice is taken out of the freezer at -18°C. How much energy is needed to turn the ice into water at room temperature (20°C)? Lfusion = 334 kJ/kg, Cice = 2.11 kJ/kg-K, Cwater = 4.19 kJ/kg.K.

Answers

The amount of energy needed to turn 1.00 kg of ice at -18°C into water at room temperature (20°C) is 370.44 kJ.  Understanding the energy transfer during phase changes is essential in various fields, including thermodynamics and engineering applications.

To calculate the energy needed to convert the ice into water, we need to consider two processes: raising the temperature of the ice from -18°C to 0°C and then melting the ice at 0°C to water at 0°C.

The energy required to raise the temperature of the ice from -18°C to 0°C:

Q1 = m * Cice * ΔT

Q1 = 1.00 kg * 2.11 kJ/kg-K * (0°C - (-18°C))

Q1 = 37.38 kJ

The energy required to melt the ice at 0°C to water at 0°C:

Q2 = m * Lfusion

Q2 = 1.00 kg * 334 kJ/kg

Q2 = 334 kJ

The energy required to raise the temperature of the water from 0°C to 20°C:

Q3 = m * Cwater * ΔT

Q3 = 1.00 kg * 4.19 kJ/kg-K * (20°C - 0°C)

Q3 = 83.8 kJ

Finally, the total energy needed is the sum of the three quantities:

Total energy = Q1 + Q2 + Q3

Total energy = 37.38 kJ + 334 kJ + 83.8 kJ

Total energy = 455.18 kJ

However, since we are starting from -18°C, we need to subtract the energy needed to raise the temperature of the ice from -18°C to 0°C, so the final answer is:

Total energy = 455.18 kJ - 37.38 kJ

Total energy = 417.8 kJ

The energy needed to turn 1.00 kg of ice at -18°C into water at room temperature (20°C) is 417.8 kJ. This calculation takes into account the energy required to raise the temperature of the ice, the energy required to melt the ice, and the energy required to raise the temperature of the resulting water. The values for specific heat capacities (Cice and Cwater) and latent heat of fusion (Lfusion) are used in the calculations to account for the specific properties of ice and water. Understanding the energy transfer during phase changes is essential in various fields, including thermodynamics and engineering applications.

To know more about energy ,visit:

https://brainly.com/question/13881533

#SPJ11

A satellite 1000 km above Mars is orbiting Mars every 15 hours.
The radius of Mars is approximately equal to 3,300 km. How far does
the satellite travel in 1 hour?

Answers

The satellite travels approximately 1,798.07 km in 1 hour. The circumference of the satellite's orbit is 8,600π km, and the satellite orbits Mars every 15 hours.

To find out how far the satellite travels in 1 hour, we need to determine its orbital circumference.

The circumference of a circular orbit can be calculated using the formula:

C = 2πr

where C is the circumference and r is the radius of the orbit.

In this case, the satellite is orbiting Mars, which has a radius of approximately 3,300 km. The satellite is 1,000 km above the surface of Mars. Therefore, the radius of the satellite's orbit is the sum of the radius of Mars and the distance above the surface:

r = 3,300 km + 1,000 km = 4,300 km

Now we can calculate the circumference:

C = 2π(4,300 km) = 8,600π km

Since the satellite orbits Mars every 15 hours, the distance traveled in 1 hour is 1/15th of the circumference:

[tex]\begin{equation}\text{Distance traveled in 1 hour} = \frac{1}{15} \cdot 8600\pi \text{ km}[/tex]

Calculating this value gives us:

Distance traveled in 1 hour ≈ 1,798.07 km

Therefore, the satellite travels approximately 1,798.07 km in 1 hour.

To know more about the circumference refer here :

https://brainly.com/question/17191505#

#SPJ11

Given vectors a=(1,5) and b=(3,-9) Find the x-component of the resultant vector: r` = 3 वे – 26 - Your Answer: Answer
Given vectors a=(-3,-3) and b=(-7,1) Find the magnitude of the resultant ve

Answers

A vector is a quantity or phenomenon that has two independent properties: magnitude and direction. Therefore, the x-component of the resultant vector is 4.

The term also denotes the mathematical or geometrical representation of such a quantity.

Examples of vectors in nature are velocity, momentum, force, electromagnetic fields and weight.

To find the x-component of the resultant vector, we need to add the x-components of vectors a and b.

Given vector a=(1,5) and vector b=(3,-9), the x-components are:

aₓ = 1

bₓ = 3

To find the x-component of the resultant vector, we add the x-components:

rₓ = aₓ + bₓ

= 1 + 3

= 4

Therefore, the x-component of the resultant vector is 4.

To know more vector:

https://brainly.com/question/7675351

#SPJ4

what volume of oxygen gas is produced when 51.0 g of mercury(ii) oxide reacts completely according to the following reaction at 25oc and 1 atm?

Answers

The given reaction is as follows;2HgO(s) → 2Hg(l) + O2(g). First of all, we need to find the moles of mercury oxide used. The molar mass of mercury oxide, HgO = 200.59 g/mol. Thus, the moles of HgO used is; mass/molar mass = 51.0 g / 200.59 g/mol = 0.2544 mol.

The stoichiometry of the given reaction tells us that the molar ratio of HgO to O2 is 2:1.

Thus, the moles of O2 produced will be 0.2544/2 = 0.1272 mol.

Now we can use the ideal gas law to calculate the volume of O2 produced.

The ideal gas law is given as; PV = nRT.

We have the following values:P = 1 atmV = unknown = 0.1272 molR = 0.0821 L atm/mol K (universal gas constant)T = 25°C = 298 K.

Now we can solve for V.V = (nRT)/P = (0.1272 mol)(0.0821 L atm/mol K)(298 K)/(1 atm) = 2.97 L.

Therefore, the volume of O2 gas produced is 2.97 L.

Learn more about ideal gas law here ;

https://brainly.com/question/30458409

#SPJ11

A ray of light is incident on a block of diamond. n= 3/2 at an
angle of 45 degree with the normal.
a) Find the angle of refraction at the boundary at A
b) Find the critical angle for the diamond
c) On

Answers

a) The angle of refraction is 28 degrees

b) The critical angle of diamond is 42 degrees

What is the critical angle?

The critical angle is a term used in optics to describe the specific angle at which light transitions from one medium to another, such as from a more optically dense medium to a less optically dense medium. It is the angle of incidence at which the refracted ray is at an angle of 90 degrees to the normal, resulting in the refracted ray being parallel to the boundary between the two media.

We know that;

n = Sin i/Sin r

n = refractive index

r = angle of refraction

i = angle of incidence

3/2 = sin 45/sin r

sinr = Sin 45 * 2/3

= 0.47

r = Sin-1 (0.47)

= 28

n = 1/Sin C

Sin C = 1/n

C = Sin-1(1/n)

C = Sin-1(1/3/2)

C = 42 degrees

Learn more about critical angle:https://brainly.com/question/30002645

#SPJ4

Two tugboats pull a disabled supertanker. Each tug exerts a constant force of 2.2x106 N, one an angle 14° west of north and the other an angle 14° east of north, as they pull the tanker a distance 0.68 km toward the north. Part A What is the total work they do on the supertanker? Express your answer in joules. DMG 195] ΑΣΦ W = Submit Provide Feedback Request Answer ? 3

Answers

The total work they do on the supertanker is 1360J.

To find the total work done by the two tugboats on the supertanker, we need to calculate the work done by each tugboat and then add them together.

The work done by a force can be calculated using the equation:

Work = Force * Displacement * cos(theta)

where:

Force is the magnitude of the force applied

Displacement is the magnitude of the displacement

theta is the angle between the force and displacement vectors

For the first tugboat:

Force = 2.2 x 10^6 N

Displacement = 0.68 km = 0.68 x 10^3 m=680m

theta = 14° west of north

Using the equation above, we can calculate the work done by the first tugboat:

Work1 = Force * Displacement * cos(theta)

For the second tugboat:

Force = 2.2 x 10^6 N

Displacement = 0.68 km = 0.68 x 10^3 m=680m

theta = 14° east of north

Similarly, we can calculate the work done by the second tugboat:

Work2 = Force * Displacement * cos(theta)

To find the total work done by the two tugboats, we can add the individual works together:

Total Work = 680+680=1360J

Learn more about Force:https://brainly.com/question/12785175

#SPJ11

Policies Current Attempt in Progress A partly full paint can has 0.842 US gallons of paint left in it. (a) What is the volume of the paint, in cubic meters? (b) if all the remaining paint is used to coat a wall evenly (wall area-145 m²), how thick is the layer of wet paint? Give your answer in meters (a) Number Units (b) Number Units eTextbook and Media GO Tutorial Attempts: 0 of 3 used S Save for Later Using multiple attempts will impact your score 15% score reduction after attempt 2 Type here to search ii 1842 AM ✔ Question 7 of 10 Current Attempt in Progress Two bicyclists, starting at the same place, are riding toward the same campground by different routes. One cyclist rides 1190 o due east and then turres due north and travels another 1410 m before reaching the campground. The second cyclist starts out by heading due north for 1910 m and then turns and heads directly toward the campground. (a) At the turning point, how far is the second cycet from the campground? (b) in what direction (measured relative to due east within the range (-180, 180) must the second cyclist head during the last part of the trip? ö O hp Type here to search F C 1047 AM Question / of 10 (a) Number (b) Number eTextbook and Media Save for Later Using multiple attempts will impact your score. 15% score reduction after attempt 2- Type here to search O ii Units Units Ĵ Attempts:0 of 3 used Submit Answer 1042 MANG

Answers

(a) The volume of paint left in the can is approximately 0.842 US gallons. Converting this volume to cubic meters, we find that it is equivalent to approximately 0.003191 cubic meters.

(b) The thickness of the wet paint layer is approximately 0.000022 meters, or 0.022 millimeters.

(a) To find the volume of paint in cubic meters, we convert the volume of paint left in the can from US gallons to cubic meters. Since 1 US gallon is equal to 0.00378541 cubic meters, we can multiply 0.842 US gallons by 0.00378541 to obtain the volume in cubic meters. This calculation gives us approximately 0.003191 cubic meters.

(b) Next, we calculate the thickness of the wet paint layer by dividing the volume of paint by the wall area. Dividing 0.003191 cubic meters by 145 square meters gives us approximately 0.000022 meters, or 0.022 millimeters, as the thickness of the wet paint layer.

To know more about cubic meters here https://brainly.com/question/30106291

#SPJ4

A teflon block of mass 5.00 kg slides to the right on a steel floor under the influence of an external applied force that is directed toward the right and has magnitude 3.00N (as you might have due to the constant pull from a cord attached to it, for instance). Enter all answers in standard units,and do not include the units in the answer fields I) Calculate the magnitude of the normal force with which the floor pushes on the block. 2 Calculate the magnitude of the frictional force acting on the block. 3Calculate the magnitude of the acceleration this block is experiencing. 4-6) Take the same problem as before and add a second external force that points in the same direction as the normal force from the floor on the block with magnitude 8.00 N. Solve the same three problems and report following the same guidance. 4) Calculate the magnitude of the normal force with which the floor pushes on che block. 5 Calculate the magnitude of the frictional force acting on the block 6Calculate the magnitude of the acceleration this block is experiencing

Answers

A Teflon block of mass 5.00 kg slides to the right on a steel floor under the influence ofan external applied force that is directed toward the right and has magnitude 3.00 N.(1)The magnitude of the normal force is 49.0 N.(2)Net force = 1.04 N.(3)The magnitude of the acceleration experienced by the block is 0.208 m/s².

(1)The magnitude of the normal force with which the floor pushes on the block is equal to the weight of the block since there is no vertical acceleration.

Weight = mass * gravitational acceleration

Weight = 5.00 kg * 9.8 m/s²

Weight = 49.0 N

Therefore, the magnitude of the normal force is 49.0 N.

The magnitude of the frictional force acting on the block can be calculated using the equation

Frictional force = coefficient of friction * normal force

Assuming a coefficient of friction of 0.04 between steel and Teflon, we can calculate:

Frictional force = 0.04 * 49.0 N

Frictional force = 1.96 N

So, the magnitude of the frictional force acting on the block is 1.96 N.

(2)  The magnitude of the acceleration experienced by the block can be calculated using Newton's second law:

Net force = mass * acceleration

The net force acting on the block is the applied force minus the frictional force:

Net force = applied force - frictional force

Net force = 3.00 N - 1.96 N

Net force = 1.04 N

(3)Now, we can calculate the acceleration:

Acceleration = Net force / mass

Acceleration = 1.04 N / 5.00 kg

Acceleration = 0.208 m/s²

So, the magnitude of the acceleration experienced by the block is 0.208 m/s².

To learn more about coefficient of friction visit: https://brainly.com/question/14121363

#SPJ11

3.Michelson interferometer a) Find the distance moved by the mirror to observe 25 counts of bright fringes given that the laser wavelength is 632.8 nm: b) If the mirror moved a distance L. produces 30

Answers

a) The distance moved by the mirror to observe 25 counts of bright fringes is 15,820 nm.

b) If the mirror moved a distance L produces 30 bright fringes, then 3L will produce 90 bright fringes.

How to solve for Michelson interferometer?

a) A Michelson interferometer is an optical instrument that uses interference to measure distances. It consists of two mirrors, a beam splitter, and a detector. The beam splitter divides the light from a laser beam into two beams, which are then reflected by the two mirrors.

The distance moved by the mirror to observe 25 counts of bright fringes is equal to 25 times the wavelength of the light. In this case, the laser wavelength is 632.8 nm, so the distance moved by the mirror is 25 × 632.8 nm = 15,820 nm.

b) The number of bright fringes produced by a Michelson interferometer is proportional to the distance moved by the mirror. In this case, the mirror moved a distance L and produced 30 bright fringes. If the mirror is moved a distance of 3L, then the number of bright fringes will be 3 × 30 = 90.

Find out more on Michelson interferometer here: https://brainly.com/question/16749752

#SPJ4

Complete question:

3.Michelson Interferometer A) Find The Distance Moved By The Mirror To Observe 25 Counts Of Bright Fringes Given That The Laser Wavelength Is 632.8 Nm: B) If The Mirror Moved A Distance L. Produces 30 Counts Of Bright Fringes, When The Mirror Was Moved By 3L How Many Counts Of Bright Fringes Would Be Observed?

B seawater velocity=1478 m/s water depth = 509 m sandstone velocity= 2793 m/s thickness 1003 m mudstone velocity= 2240 m/s thickness = 373 m Air Gun Energy Source 9° * Note: Illustration is not to scale. Hydrophone Receivers seafloor sand/mud 3. How long does it take for energy to travel directly from the air gun to the first hydrophone (no bounces)?

Answers

it takes 0.70 seconds for energy to travel directly from the air gun to the first hydrophone (no bounces).

Given,

B seawater velocity = 1478 m/s

water depth = 509 m

sandstone velocity = 2793 m/s

thickness = 1003 mmudstone

velocity = 2240 m/s

thickness = 373 m

Energy source is Air GunAngle of incidence = 9°

Let's calculate the time taken for energy to travel directly from the air gun to the first hydrophone (no bounces).

From the given information, we can calculate the distance traveled by energy.Let's calculate the distance travelled by the energy to reach the first hydrophoneDistance travelled in water (d1) = velocity * time takenLet, t1 be the time taken to reach the seafloor

t1 = (2 * depth) / Bw= (2 * 509) / 1478w = 1.37 s

Distance travelled in sandstone (d2) = velocity * time takenLet, t2 be the time taken to travel through sandstone t2 = thickness / Vs= 1003 / 2793t2 = 0.359 s

Distance travelled in mudstone (d3) = velocity * time takenLet, t3 be the time taken to travel through mudstone t3 = thickness / Vm= 373 / 2240t3 = 0.166 s

Let's calculate the distance travelled by the energy to reach the first hydrophoneD1= sin(9°) * Dwhere, D is the distance between the air gun and the first hydrophone.

Using Pythagoras theorem, we can find the distance D.

D² = d1² + d2² + d3² + D1²Now, D = sqrt(d1² + d2² + d3² + D1²)

Time taken to travel directly from the air gun to the first hydrophone (no bounces) = (D1) / (1478) + (D2) / (2793) + (D3) / (2240)

Where D1, D2 and D3 are calculated asD1= sin(9°) * DD² = d1² + d2² + d3² + D1²D = 6560 mD1 = 1034.5 mD2 = D3 = 0By substituting the values, we get

Time taken to travel directly from the air gun to the first hydrophone (no bounces) = (D1) / (1478) + (D2) / (2793) + (D3) / (2240)= 0.70 s

Therefore, it takes 0.70 seconds for energy to travel directly from the air gun to the first hydrophone (no bounces).

learn more about hydrophone here

https://brainly.com/question/2189435

#SPJ11

The change in momentum that occurs when a 1kg ball traveling at
4m/s strikes a wall and bounces back at 2m/s is
A) 2 kg m/s
B) 4 kg m/s
C) 6 kg m/s
D) 8 kg m/s

Answers

When a 1 kg ball moving at 4 m/s strikes a wall and bounces back at 2 m/s, the change in momentum is 6 kg m/s. The correct answer is option C.

Momentum is defined as the product of mass and velocity, and it is a vector quantity. When an object strikes another object, there is a change in momentum. The law of conservation of momentum states that the total momentum of a system of objects remains constant if there are no external forces acting on the system.

The change in momentum of an object that strikes a stationary wall and bounces back with the same velocity is given by:
Δp = 2mv
Where,

m is the mass of the object,

v is the velocity of the object after the collision.

In this case, the ball has a mass of 1 kg and an initial velocity of 4 m/s. After bouncing back from the wall, its velocity is 2 m/s. Therefore, the change in momentum is:
Δp = 2mv = 2(1 kg)(2 m/s - 4 m/s) = -4 kg m/s

The negative sign indicates that the momentum is in the opposite direction to the initial momentum. To get the absolute value of the change in momentum, we take the magnitude, which is 4 kg m/s. However, since we are interested in the change in momentum, we need to include the direction, which is opposite to the initial momentum.

Therefore, the change in momentum that occurs when a 1 kg ball traveling at 4 m/s strikes a wall and bounces back at 2 m/s is 6 kg m/s, and the correct option is C.

Learn more about momentum:

https://brainly.com/question/18798405

#SPJ11

a baseball m=145g traveling 32 m/s moves backwards 25cm when the ball is caught. what is the force exerted by the ball on the glove

Answers

The force exerted by the ball on the glove is approximately 148.48 N. Note that the negative sign indicates that the force is exerted in the opposite direction of the ball's motion.

When the ball is caught, the force exerted by the ball on the glove can be calculated using the formula F = ma, where F is the force, m is the mass of the baseball, and a is the acceleration. Since the ball is initially traveling forward and then suddenly moves backward upon contact with the glove, its change in velocity can be calculated using the formula Δv = vf - vi, where Δv is the change in velocity, vf is the final velocity (0 m/s), and vi is the initial velocity (32 m/s). Therefore, Δv = 0 - 32 = -32 m/s.Using the equation for acceleration, a = Δv/t, where t is the time interval for the ball to come to rest, we can solve for a.

To solve for the force, we need to first calculate the time interval Δt. Since we know the initial velocity of the ball, we can use the formula for average velocity, vavg = (vi + vf) / 2, to calculate the average velocity of the ball over the time interval Δt. Solving for Δt, we get:Δt = (2d) / vavgwhere d is the distance the ball moves backward (25 cm = 0.25 m).vavg = (vi + vf) / 2vavg = (32 + 0) / 2vavg = 16 m/sΔt = (2d) / vavgΔt = (2 * 0.25) / 16Δt = 0.03125 sNow that we have Δt, we can solve for the force using the formula for impulse:J = FΔtJ = mΔvJ = (0.145 kg)(-32 m/s)J = -4.64 N-sJ = FΔt-4.64 N-s = F(0.03125 s)F = -4.64 N-s / 0.03125 sF = -148.48 N

The force exerted by the ball on the glove is approximately 148.48 N. Note that the negative sign indicates that the force is exerted in the opposite direction of the ball's motion.

To learn more about Force visit;

https://brainly.com/question/13191643

#SPJ11

Other Questions
the clash of ethical systems between government and business is centered around Given B = 12.0 b = 7.02 a = 8.44, use the Law of Sines to find the remaining sides and angles of the triangle. You should use a calculator for this question. (6 points) Given a 10, b=8, and c=13, use the Law of Cosines to solve the triangle for the value of angle A. You should use a calculator for this question. (4 points) What word describes when water is attracted to other substances?a.cohesionb.adhesionc.capillary actiond.surface tension In the context of Basel I, what is the Minimum Capital that must be prepared by a bank providing credit? corporation Rp. 600 billion, individual credit without collateral Rp. 200 billion and home mortgage Rp. 2.2 Trillion? Dan McClure owns a thriving independent bookstore in artsy New Hope, Pennsylvania. He must decide how many copies to order of a new book, Power and Self-Destruction, an expose on a famous politician's lurid affairs. Interest in the book will be intense at first and then fizzle quickly as attention turns to other celebrities. The book's retail price is $ 20 and the wholesale price is $12. The publisher will buy back the retailer's leftover copies at a full refund, but McClure Books incurs $4 in shipping and handling costs for each book returned to the publisher. Dan believes his demand forecast can be represented by a normal distribution with mean 200 and standard deviation 80. a. Dan will consider this book to be a blockbuster for him if it sells more than 400 units. What is the probability Power and Self-Destruction will be a blockbuster? b. Dan considers a book a "dog" if it sells less than 50% of his mean forecast. What is the probability this expose is a "dog"? c. What is the probability demand for this book will be within 20% of the mean forecast? d. What order quantity maximizes Dan's expected profit? e. How many books should Dan order if he wants to achieve a 95% in-stock probability? f. Being an introspective chap, Dan decided to evaluate his forecasting skills in his spare time. He collected the following data on recent books he felt matched the characteristics of Power and Self-Destruction. If Dan used these data to construct an empirical distribution function, then what would be his optimal order quantity? (Assume Dan's initial forecast is 200 units.) Diamond Drillers purchased a mining site for $550,000 on August 1, 20Y8. The company expects to mine ore for the next 10 years and anticipates that a total of 70,000 tons will be recovered. During 20Y8, the company extracted 6,200 tons of ore. The depletion expense for the year 2018 is O a. b. $22,917 O b. a. $55,000 O c. d. $48,714 O d.c. $4,871 Question 5 (1 point) Countries or regions that have greater wealth... O tend to have a higher total fertility rate O tend to be no different than countries with less wealth O tend to have a lower total fertility rate O none of these are true since it's the number of males that really determine a population's growth rate Emma works at a clothing store, and works on commission.The amount she makes each week depends on how much clothing she sells, and she also receives a weekly amount regardless of her sales.Emma also has to pay for her meals during her shifts, which detracts from her income.Emmas income can be modelled by the following equation: I=100+12J+5T+7S+5H-8M where I is her income, J is the number of jeans she sells, T is the number of T-shirts she sells, S is the number of shorts she sells, H is the number of hats she sells, and M is the number of meals she buys.If Emma makes $205 in a week and sells: 6 pairs of jeans, 4 shorts, and 5 hats, and buys 5 of her meals.How many T-shirts did she sell? There are two competing spas in town that offer 1-hour massages, pedicures, and manicures at the following prices. Massage Perdicure ManicureCalm Retreat [$70 $30 $251] Lotus Spa [$90 $36 $27] a) If Janet takes her friend to the Lotus Spa and has 4 manicures done, how much will it cost? [DOK 3: 3 marks] b) What is the difference in the price between a massage at the Lotus Spa and one at the Calm Retreat? [DOK 3:3 marks] If your views about what the $/ is going to do are mixed, whichdo you think would be a better fit? Show that the vector-valued function shown below describes the motion of a particle moving in a circle of radius 1 centered at the point (2, 2, 1) and lying in the plane 2x + 2y - 4z = 4; r(t) =(2i + 2j + k) + cos t [1/rad2 i - 1/rad2 j] + sin t [1/rad3 i + 1/rad3 j + 1/rad3 k]. Write 3 parametric equations for x, y, and z explain at least 3 companies apple,Samsung tesla that successfully implemented six sigma. What are the main results? (Summarize your answers within one page of A4 paper) please answer the questions as soon as possible discrete matha) Draw the Hasse diagram for the poset divides (1) on S={2,3,6,8,12, 24) b) Identify the minimal, maximal, least and greatest elements of the above Hasse diagram "Grexit, Brexit, Trump, referenda, and the rise of populism in the European Union. 2016 was an uncertain and turbulent year in terms of political risk. For companies and investors, existing economic trends suddenly became less certain. Political risk is a term that is not regularly encountered in the risk frameworks of financial institutions. However, unexpected changes in policies could significantly impact existing business models. A hotly debated topic is to what extent the development of digitalization and policies of globalization have contributed to the emerging political risk in Western democracies, and how companies could anticipate such risk. (Source by Articles of The Impact of Digitalization and Globalization on Political Risk) Based on the statement above, explain FIVE (5) strategies to manage political risk. If we change our credit terms by increasing the discount, we would expect the ACP to You measure 27 textbooks' weights, and find they have a mean weight of 41 ounces. Assume the population standard deviation is 11.1 ounces. Based on this, construct a 90% confidence interval for the true population mean textbook weight. Give your answers as decimals, to two places Canada Corporation began 2021 with retained earnings of $210 million. Revenues during the year were $520 million, and expenses totaled $360 million. Canada declared dividends of $59 million. What was the company's ending balance of retained earnings? 10.19a. From the information given here determinethe 95% confidence interval estimate of the popula-tion mean.x =100 = 20 n = 25b. Repeat part (a) with x =200.c. Repeat part (a) with x =50 Positive action is the only way to ensure an increase in diversity. Discuss". Critically assess this statement, illustrating it with a practical example. 500 words Amy plants to buy watermelon in a supermarket she found that the average weight of the watermelon is 20 pounds with a standard deviation of 5 pounds what is the mean and standard deviation of the sampling distribution of the sample mean a sample of 5 watermelon